1
$\begingroup$

Is it possible to transform this equation to give R? $$y=x\left[\frac{\left(1+\frac{R}{12}\right)^{12\times{25}}}{\frac{R}{12}}-1\right]$$

  • 0
    I've edited your question. Please check whether it is what you meant to write. I appreciated your handwork, but please use LaTeX in future :)2012-10-09
  • 0
    For some basic information about writing math at this site see e.g. [here](http://meta.math.stackexchange.com/questions/5020/), [here](http://meta.stackexchange.com/a/70559/155238), [here](http://meta.math.stackexchange.com/questions/1773/) and [here](http://math.stackexchange.com/editing-help#latex).2012-10-09
  • 0
    related: http://math.stackexchange.com/questions/207864/how-to-solve-for-i-and-n-in-compound-interest-formula2012-10-09

2 Answers 2